Gravigyro-Magnetic Equations with the Angle Between Spin States and a Force Equation

Discussion in 'Pseudoscience Archive' started by Reiku, Feb 28, 2012.

Thread Status:
Not open for further replies.
  1. Reiku Banned Banned

    Messages:
    11,238
    ''I know enough physics that you can't try that argument against me and you ignore me too. And your "I'll go find a video!" thing always backfires. It either shows you're just mindlessly copying from someone, as you copied Susskind in your discussion with James, or you're actually mistakenly trying to paraphrase source material you don't understand. ''


    oh... mmm... you're referring to the time I said about physics having ''something to say about perpendicular and orthogonality.''

    I was right. I just never specified it was pointers it spoke about. My definitions where correct though, whilst people like cpt Bork seemed to not even recognize this literature, saying I was making things up, or something along those effects.
     
  2. Google AdSense Guest Advertisement



    to hide all adverts.
  3. Reiku Banned Banned

    Messages:
    11,238
    ... and I will find the video. I have come to one that is close in nature to it, so it cannot be far. All I do is copy as I see it being taught. You cannot get more accurate than that.
     
  4. Google AdSense Guest Advertisement



    to hide all adverts.
  5. Balerion Banned Banned

    Messages:
    8,596
    That sounds about right.
     
  6. Google AdSense Guest Advertisement



    to hide all adverts.
  7. Reiku Banned Banned

    Messages:
    11,238
    It's driving me insane looking for this, so I'll just add more from my own work and show you another work which is (very similar) to deriving the angle in respects to the approach I gave; in fact, we will start with the latter. I did a search and found the following:

    usimg the projection of both we can draw the vectors and we can find the angles between them

    i was just wondering if there was a way of doing this using \( \vec{L} \cdot \vec{S} \)

    \( L \cdot S = |L| |S| \cos \theta \)

    http://www.physicsforums.com/showthread.php?t=164262

    As you can see, he finds a quick way to derive the angle (obviously my case was a by a far an in-depth calculation for the angle between the spins in a quantum sense, since we are specifically using pauli matrices, but the same mathematical principle is behind it.

    Even though I have had problems finding the video, I can justify many of the terms. Hopefully I won't need to justify things like \((\hat{n} \cdot \sigma)\) but maybe things like:

    \(\begin{bmatrix}\ (n_3) & (n_{-}) \\ (n_{+}) & (-n_3) \end{bmatrix} \begin{bmatrix}\ \alpha \\ \beta \end{bmatrix} = \begin{bmatrix}\ \alpha \\ \beta \end{bmatrix}\)

    Well draw your attention please to this following paper; took me a while even to find a relevant paper to all this work, but after typing in enough buzzwords, I did find one:

    http://www.tcm.phy.cam.ac.uk/~bds10/aqp/handout_spin.pdf

    You will see that matrix in there as well, and it properly justifies the column

    \(\begin{bmatrix}\ \alpha \\ \beta \end{bmatrix}\)

    Indeed then that]

    \((\sigma \cdot n)^2 = 1\)

    Is true. So as you can see, I have found a very relevant paper to all this stuff. Sure I cannot find any citations proving the angle-derivation, I won't be able to do that until I find the video, but surely by now the work is beginning to look more credible. Funkstar was well out of his depth in coming in here foolishly as branding it as crap, as he clearly doesn't understand any of it.
     
    Last edited: Mar 2, 2012
  8. Reiku Banned Banned

    Messages:
    11,238
    In fact, I think this citation http://www.tcm.phy.cam.ac.uk/~bds10/aqp/handout_spin.pdf will help me develope new ways of understanding the range of possibilities for my equation. I've only had a quick glance over it and already I am learning a great deal more about these processes.
     
  9. Reiku Banned Banned

    Messages:
    11,238
    \(\begin{bmatrix}\ \alpha \\ \beta \end{bmatrix}\)

    So as you will see, this is really a ''linear yet general state of spin''.

    Let me refresh your memory on the force equation

    \(F_{ij} = -\frac{\partial V(r_ij)}{\partial r_{ij}}\hat{n}_{ij}\)

    This means if you involve spin like I had as a dot product on the unit vector and square everything you really have

    \(F_{ij} = -\frac{\partial V(r_ij)}{\partial r_{ij}}\hat{n}_{ij}\)

    \(-\frac{\partial^2 V^2 (r_{ij})^2}{\partial^2 r^{2}_{ij}} \mu(\hat{n} \cdot \vec{\sigma}_{ij})^2 = -\frac{\partial^2 V^2 (r_{ij})^2}{\partial^2 r^{2}_{ij}} \begin{bmatrix}\ \mu(n_3) & \mu(n_{-}) \\ \mu(n_{+}) & \mu(-n_3) \end{bmatrix}^2\)

    \(= (-\frac{\partial V(r_ij)}{\partial r_{ij}})^2 \mathbf{I}\)

    \(= F_{ij}^{2}\)

    Where of course, the \(1\) before is simply the identity matrix thus I have used this notation instead because it is more recognizable.
     
    Last edited: Mar 2, 2012
  10. Reiku Banned Banned

    Messages:
    11,238
    Keep in mind also that

    \(\alpha* \alpha + \beta* \beta = 1\)
     
  11. Reiku Banned Banned

    Messages:
    11,238
    So you may also derive

    \(-\frac{\partial^2 V^2 (r_{ij})^2}{\partial^2 r^{2}_{ij}} \mu(\hat{n} \cdot \vec{\sigma}_{ij})^2\begin{bmatrix}\ \alpha \\ \beta \end{bmatrix}^2 = -\frac{\partial^2 V^2 (r_{ij})^2}{\partial^2 r^{2}_{ij}} \begin{bmatrix}\ \mu(n_3) & \mu(n_{-}) \\ \mu(n_{+}) & \mu(-n_3) \end{bmatrix}^2\begin{bmatrix}\ \alpha \\ \beta \end{bmatrix}^2\)

    And it still spits out, I think if I have read this right

    \(= (-\frac{\partial V(r_ij)}{\partial r_{ij}})^2 \mathbf{I}\)
     
  12. Reiku Banned Banned

    Messages:
    11,238
    A different insight, but not far off my own notation for magnetic moments along a spin given before as \(\mu(\sigma \cdot n)\), the gyromagnetic ratio (perfect name to the title), is used in the equations in that specific link.

    Thus we can now see our equations in a completely new light, which I will write up now in the following post.
     
  13. Reiku Banned Banned

    Messages:
    11,238
    \(-\frac{\partial^2 V^2 (r_{ij})^2}{\partial^2 r^{2}_{ij}}\frac{g e}{2Mc} (\hat{n} \cdot \vec{\sigma}_{ij})^2 = -\frac{\partial^2 V^2 (r_{ij})^2}{\partial^2 r^{2}_{ij}} \begin{bmatrix}\ g \gamma(n_3) & g \gamma(n_{-}) \\ g \gamma(n_{+}) & g \gamma(-n_3) \end{bmatrix}^2\)
     
  14. Reiku Banned Banned

    Messages:
    11,238
    Where \(g\) is the g-factor and the gyromagnetic ratio is \(\gamma = \frac{e}{2Mc}\), should have mentioned it.
     
  15. khan Registered Senior Member

    Messages:
    130
    I have only been studying classical physics and general relativity so far because Feynman said nobody understands quantum mechanics. Your equation is different than the one in the spin handout pdf you linked to.

    http://www.tcm.phy.cam.ac.uk/~bds10/aqp/handout_spin.pdf

    Can you explain all the terms in your equation in the quote to me? Pretend I am a time traveler from 1915, before the era of 1920's physics

    Please Register or Log in to view the hidden image!

     
  16. AlphaNumeric Fully ionized Registered Senior Member

    Messages:
    6,702
    So you admit your excuse was completely invalid, you're aware it isn't a blanket statement? So why did you make it? Once again you clutch at straws and only admit your inaccuracies when you're called on them.

    Progressively odd? Please explain how it is odd? I've outlined your mistakes and pointed out your dishonesty. You're the one repeatedly lying, knowingly, to people who openly say "We don't believe you". That would be 'progressively odd' if it were progressive. It just seems to be who you are.

    As for being aggressive, I'm tired of your shit.

    Then you have a terrible teacher.

    You think that's the only instance? This is just yet another example of you being stupid enough to lie to me about something I said to you! Seriously, that's stupid. I really, really hope you're knowingly doing this, that this is all a massive exercise in trolling, because the alternative is you really believe your own nonsense.

    Hell, I even explicitly said your discussion with James. Remember how you were posting stuff about Hamiltonians and quantum field theory and trying to explain things to James, only for it to then transpire your were mistakenly parroting Susskind, right down to his dubious notation?

    It doesn't mean you understand it. I can write Chinese if someone puts a Chinese newspaper in front of me, doesn't mean I understand it. My father used to have to give speeches in Welsh as part of his job, except he can't speak Welsh so colleagues would teach him how to say everything like a parrot. Heck, parrots can speak English but do they understand it? Nope.

    It's a form of plagiarism, passing off someone else's explanations as your own. You aren't formulating your own explanations using your own understanding, you just search for particular words to match and then to reproduce the relevant section. It's how laughably stupid chat bots work.

    It's not clear whether he's saying the mathematics is bull or whether your attempts to present yourself as understanding it is bull. The latter is most definitely true, you don't understand this stuff. The fact you've had to hunt around for material to back you up, rather than just knowing a book its in demonstrates that.

    No one believes your claim you understand this. Your repeated dishonest and misrepresentation of other people's explanations, other people's bookwork transcribed directly, is plagiarism. Your blanket parroting of Susskind is multiple instances of that. If you're spending 4 hours a day doing this stuff then you're wasting your life. 5 years of 4 hours a day is more work than most people who do a degree yet you couldn't pass an A Level exam!
     
  17. Reiku Banned Banned

    Messages:
    11,238
    Good, your tired of my shit and I'm tired of yours.

    Can I take this as a card now to block you?
     
  18. Reiku Banned Banned

    Messages:
    11,238
    Sure. The longer terms written out in that matrix are simply

    \(n_1 - in_2 = n_{-}\)

    \(n_1 + in_2 = n_{+}\)

    And any subscripts like \(z\) or \(x\) or \(y\) purport to the spin directionality. Does this make sense now Khan?
     
  19. AlphaNumeric Fully ionized Registered Senior Member

    Messages:
    6,702
    I'm tired of your* shit but I'm not going to sit idly by while you liar and plagiarise. If you really understand this stuff you'd be able to step up when I challenge you to show something or justify something. You never do, usually because the video you're plagiarising doesn't give the explanation, such as in the case of Susskind's comment about time independence of L=T-V implying T+V is time independent but he didn't prove it, which you parroted and couldn't explain.

    If you don't like people slating you then you should be more honest. If you were asking questions about basic calculus and linear algebra I'd not have an issue. Instead you're proclaiming things about advanced topics. You failed James's challenge miserably, despite the questions being on subjects you should know inside out to do the stuff you spout crap about.

    I can understand why you want to put me on block. Be reminded of your many short comings and obvious mistakes must get in the way of drinking your own koolaid.

    * Note the difference between you're tired of my shit and I'm tired of yours. That's the second time you've gotten your/you're wrong in the last day or so. Your grammar is improving at the same rate as your physics.
     
  20. Reiku Banned Banned

    Messages:
    11,238
    Don't worry AN.

    You know, I actually love physics that much, it actually hurts when someone calls me a plaigarist of physics. If my ability to write about physics is really that SHIT, that I am having problems distinguishing my work from work which isn't mine, then I assure you, it is completely unintentional.

    Just like when you said I plaigarised my tachyonic field equations. I didn't, in fact you brought it up again recently saying, ''but all you did was copy standard equation''.

    WRONG

    I actually made it clear from the very beginning of the post that those equations where extensions of Tsao Changs work on the Dirac Equation, thus I was carrying on the work to make a more complete theory... along with a theory on the Higgs Boson.

    I also said to you that from now on I would make my equations well-known so there cannot be any confusion. Well, James gave me a brilliant idea when he started to show me what the difficulties where. I actually appreciated that a LOT MORE than simply banning me. From now on, I am just going to post a list of equations every time I make a thread like that (specifying) which equations are EXACTLY mine.

    That should shut your cake hole in the future, especially if you are feeling so strong about this, that you cannot help yourself but involve yourself in my threads.

    Now can I block you?
     
  21. Reiku Banned Banned

    Messages:
    11,238
    And fuck my grammar. I just woke up.
     
  22. Reiku Banned Banned

    Messages:
    11,238
    So? It would have been catagorically worse if I had got it wrong. There loads of things in physics I have ''heard'' about but never in practice worked out. Plenty.

    And if you sit there and you're not the same,you'd be a liar, for sure.
     
  23. James R Just this guy, you know? Staff Member

    Messages:
    39,397
    Reiku:

    I'm confused.

    What use is any of your "work" in this thread?

    In other words, what problem are you solving here? Is this work novel? Does it address an outstanding problem in physics? Or does it dispute existing ideas?

    Could you please explain exactly where the work in this thread departs from the work of others, and why it is significant?

    Thanks.
     
Thread Status:
Not open for further replies.

Share This Page